Unique Decomposition of Elements in an Abelian Group

Click For Summary
The discussion revolves around proving that every element in an abelian group A can be uniquely expressed as a sum of elements from two subgroups A_r and A_s, where n is the exponent of the group and can be factored into relatively prime integers r and s. The key steps involve demonstrating that A = A_r + A_s and that their intersection is trivial, which is necessary for establishing that A is the direct sum of these subgroups. A proof attempt is made to show that A_r is a subgroup and to explore the implications of the smallest exponent assumption. The participant reflects on the challenges of self-studying algebra and the importance of clarifying assumptions in their approach. Ultimately, they find clarity in their reasoning through the process of writing out their thoughts.
UNChaneul
Messages
18
Reaction score
0

Homework Statement


Let A be an abelian group, written additively, and let n be a positive integer such that nx=0 for all x \in A. Such an integer n is called an exponent for A. Assume that we can write n=rs, where r, s are positive relatively prime integers. Let A_{r} consist of all x \in A such that rx=0, and similarly A_{s} consist of all x \in A such that sx=0. Show that every element a \in A can be written uniquely in the form a=b+c, with b \in A_{r}, and c \in A_{s}. Hence A=A_{r} \oplus A_{s}.


Homework Equations



Theorem #1
The abelian group A is a direct sum of subgroups B and C if and only if A=B+C and B \cap C = {0}. This is the case if and only if the map B*C \to A given by (b,c) \mapsto b+c is an isomorphism.


The Attempt at a Solution


So essentially, what needs to be shown is that A=A_{r}+A_{s} and that A_{r} \cap A_{s} = {0}. I went ahead and used the latter form of Theorem #1, namely showing that A_{r} * A_{s} \to A is an isomorphism. I already showed that A_{r} * A_{s} is a homomorphism and that it is injective. So, the last step is to show that it is also surjective in order to establish that it is an isomorphism.

Conjecture #1: A_{r} is a subgroup of A with order r.
Proof: We know that 0 \in A_{r} since r0=0. Suppose a_{1},a_{2} \in A_{r}. Then r(a_{1}+a_{2})=r a_{1} + r a_{2} = 0 since r is some positive integer (not necessarily in A). It also follows that -a_{1} \in A_{r} since r(-a_{1})=0. Therefore A_{r} is a subgroup of A.

This next part of the proof of the conjecture is what I am concerned about. Suppose m \in A_{r} such that m \neq 0. We know m exists since for some y \in A, rsy=ny=0, so m=sy is one possible choice, assuming sy \neq 0 for at least one y. This is clearly the case or we can continue down by descent, replacing n with s in the problem. Now out of the possible choices for m, choose the smallest one. I think m generates A_{r}, but I am not sure how to prove this. Maybe it isn't even a generator, so the entire method is flawed.

However, if it is, then the rest of the proof follows, since we can then show that since we would know m generates A_{r} and rm=0, so m has period r. It then follows that since A_{r} \cap A_{s} = {0}, and A_{r} * A_{s} has order rs=n, A_{r}*A_{s} must be surjective onto A since A also has order n, and A_{r}, A_{s} were shown to be subgroups of A.

This is using the fact that the order of A should be n since nx=0 for all x \in A, which includes for its generators.


The joys of self studying algebra :D
 
Physics news on Phys.org
Now that I think about I think I implicitly assumed that we are using the smallest possible exponent n, so things are awkward if we are not... for instance if A were the group who elements you get by addition modulo 6. We could use n=6, n=12,..., and I assumed n=6 would be the choice. So that is another problem. Makes me think I am approaching the problem the wrong way.
 
Last edited:
Nevermind, solved my own problem. Guess writing things up in a different form can be useful haha.
 
Question: A clock's minute hand has length 4 and its hour hand has length 3. What is the distance between the tips at the moment when it is increasing most rapidly?(Putnam Exam Question) Answer: Making assumption that both the hands moves at constant angular velocities, the answer is ## \sqrt{7} .## But don't you think this assumption is somewhat doubtful and wrong?

Similar threads

  • · Replies 2 ·
Replies
2
Views
2K
Replies
5
Views
3K
  • · Replies 13 ·
Replies
13
Views
4K
  • · Replies 4 ·
Replies
4
Views
2K
Replies
9
Views
2K
Replies
3
Views
2K
  • · Replies 11 ·
Replies
11
Views
2K
  • · Replies 31 ·
2
Replies
31
Views
2K
  • · Replies 6 ·
Replies
6
Views
3K
Replies
3
Views
1K